PT72.S3.Q7 - In one study hospital patients' immune system

sandypantssandypants Alum Member
edited June 2018 in Logical Reasoning 231 karma

I'm not sure if this is the correct place? Are we allowed to ask questions about individual questions? If it's not allowed (sorry!) let me know and I will delete this thread, or someone can take appropriate action.

Assuming this is all kosher, I don't understand why the answer is A for this question. This was one of those questions where I thought all of the answer choices seemed wrong--I was stuck between B and D, and didn't even consider A. I ruled out A because I thought the stimulus was talking about a hypothetical situation where the patients with a greater tendency to laugh weren't laughing a lot, so why would the answer be that it doesn't take into account that they laughed more?

Is A correct because when people, whose tendencies to laugh are greater, "laugh a little" they laugh MORE than other patients who laugh a greater amount than usual, but still less than the laughy people laughing less? (I thought this greater amount meant that they were laughing more than the people who have the tendency to laugh more. Is it that they are laughing a greater amount than they usually laugh? If so then I guess I understand A...)

Sorry this isn't super succinct but I am hella confused, any help would be appreciated!

Admin note: edited title
https://7sage.com/lsat_explanations/lsat-72-section-3-question-07/

Poll
  1. Do you hate when your feet get wet in the rain?8 votes
    1. Yes
      50.00%
    2. No
        0.00%
    3. I don't go outside, LSAT studies only!!
      50.00%

Comments

  • akistotleakistotle Member 🍌🍌
    9377 karma

    This one is pretty tough.

    [Context]
    Comic Videos ==correlation== Immune Systems Stronger
    ———
    Laughter ==cause==> Recovery from Illness

    [Argument]
    Tendency to Laugh ==correlation== Immune Systems Stronger
    ———
    Recovery from Illness: High Tendency to Laugh (Laugh a Little) > Low Tendency to Laugh (Laugh a Lot)

    We just have to accept the context as a fact (although it has a possible correlation-causation flaw). From the context, we know that laughter causes recovery from illness.

    So [Tendency to Laugh ==correlation== Immune Systems Stronger] can be explained by a different hypothesis, and based on the context, it's more reasonable to assume [High Tendency to Laugh] laugh more, which causes recovery from illness.

  • sandypantssandypants Alum Member
    edited July 2018 231 karma

    @akistotle

    OMG I just looked in my drafts and realized I didn't post my comment from when you replied T_T I'm so sorry, you are SO appreciated and thank you so much for taking the time to help me.

    Here is what I meant to reply to you weeks ago

    Wow, thank you so much! You're brilliant! This took a bit for me to process but is much more clear and makes so much more sense than what I was thinking. I'll definitely have to come back to this question again!

  • akistotleakistotle Member 🍌🍌
    9377 karma

    @sandypants said:
    @akistotle

    OMG I just looked in my drafts and realized I didn't post my comment from when you replied T_T I'm so sorry, you are SO appreciated and thank you so much for taking the time to help me.

    Here is what I meant to reply to you weeks ago

    Wow, thank you so much! You're brilliant! This took a bit for me to process but is much more clear and makes so much more sense than what I was thinking. I'll definitely have to come back to this question again!

    Haha! No worries ;) and you are welcome!

  • JustDoItJustDoIt Alum Member
    3112 karma

    @akistotle said:
    This one is pretty tough.

    Top three hardest LR questions of all time if you ask me! Great explanation @akistotle!

  • MambaMentality93MambaMentality93 Live Member
    503 karma

    I'm going to have to ask y'all to explain this to me like I'm a 3 yo because there is an element that I just cannot get through my thick head.
    Our role in flaw questions is to weaken the link between P and C. But in this question, the right answer attacks/contradicts the conclusion. Why? Aren't we supposed to not attack Goku or the car but instead focus on the beam?
    We "overlook the possibility that whose tendency to laugh was greater to begin with laughed more at the comic videos than did other patients," because the conclusion of the argument clearly states that patients with a greater tendency to laugh are helped MORE EVEN IF they laughed a little!

  • yunonsieyunonsie Member
    611 karma

    I think the argument makes an assumption when it uses 'much greater immune system strength occurred in the patients whose tendency to laugh was greater to begin with' as a premise for the conclusion.

    In order to arrive at that comparative conclusion, the author assumes that the 'much greater immune system strength' comes from the fact that this group of people is predisposed to laughing. Why? Because the conclusion states that this group, just by laughing a little, would improve more than other patients who are laughing a lot. So it's not the actual laughing that is being attributed (unfortunately a very important distinction).

    In doing so, the argument fails to consider the possibility that the greater strength is caused the amount of laughter that these two groups are likely to produce and NOT the disposition to laugh. We can also confirm earlier in the stim that laughter positively correlates with recovery as like a general rule of thumb for this question.

    Hopefully that helps! If it doesn't... I can try and explain it a different way cuz there are a lot of parts to this @KingTChalla

  • Help2222Help2222 Member
    240 karma

    I think this was great question to ask for help about. I too was strongly considering B and D. But since the conclusion was a comparative answer A does compare and judges between them. thanks again!

  • MambaMentality93MambaMentality93 Live Member
    503 karma

    @yunonsie thanks so much, mate!!!!

Sign In or Register to comment.